PRACTICE Describe the domain and range of each function using inequality or set-builder notation.​

PRACTICE Describe The Domain And Range Of Each Function Using Inequality Or Set-builder Notation.

Answers

Answer 1

In the picture there are 4 graphs.

1st graph we get domain as (-∞,+∞) and range as (-∞,+∞).

2nd graph we get domain as [-4,4] and range as [3,-1].

3rd graph we get domain as [-4,1] and range as [1,-4].

4th graph we get domain as (-∞,4] and range as (-∞,-2].

Given that,

In the picture there are 4 graphs.

We have to find the domain and range of the each function using inequality or set-builder notation.

A function's domain is the set of values that we are allowed to enter into our function.

This set consists of the values of the x-axis for a function like f. (x).

The range of values that can be used as input for a function is called this. The values in this set hence, once we input an x value, the function returns. The y values are those.

1st graph we get domain as (-∞,+∞) and range as (-∞,+∞).

2nd graph we get domain as [-4,4] and range as [3,-1].

3rd graph we get domain as [-4,1] and range as [1,-4].

4th graph we get domain as (-∞,4] and range as (-∞,-2].

To learn more about domain visit: https://brainly.com/question/28135761

#SPJ1


Related Questions

What is the value of 2^4 + 9x + 2 when x= 2?​

Answers

The value of 2⁴ + 9x + 2 when x = 2  is 36.

What is an Algebraic Expression?An algebraic expression in mathematics is one that contains variables, constants, and algebraic operations (addition, subtraction, etc.). Terms comprise expressions. These expressions use unknown coefficients, constants, and variables. These three terms combined are referred to as an expression.It should be noted that an algebraic expression lacks sides and the equal sign, in contrast to an algebraic equation.You must substitute a number for each variable and carry out the arithmetic operations in order to evaluate an algebraic expression.

Given:

Algebraic expression is 2⁴ + 9x + 2

Where x = 2

So substituting the value of x to find the result of the expression,

2⁴ + 9x + 2 = 16 + 9×2 + 2 = 16 + 18 + 2 = 36

Hence the value of the expression is 36.

To learn more about algebraic expression visit:

https://brainly.com/question/7463555

#SPJ9

Jordon is calculating the slope of the trend in the scatter plot below. How can Jordon use the plot to check the reasonableness of his answer

Answers

Answer: His slope should be positive because the trend line is slanted up to the right.

Step-by-step explanation:

This answer is correct because the third line is slanted up to the right, so it makes it positive.

Please help me asap!! I will try an give brainliest to the person who has the correct answer! (The equation that I answered in the picture was on accident, I'm not sure if that is the actual answer.)

Thank you!

Answers

Answer:

Your answer would be the last one since that's the one that doesn't have a changed result when the numbers are moved around.

To pay for a home improvement project that totals $16,000, Genesis is choosing between taking out a simple interest bank loan at 8% for 3 years or paying with a credit card that compounds monthly at an annual rate of 15% for 7 years. Which plan would give Genesis the lowest monthly payment?

Answers

well, let's first check with the Bank

[tex]~~~~~~ \textit{Simple Interest Earned Amount} \\\\ A=P(1+rt)\qquad \begin{cases} A=\textit{accumulated amount}\\ P=\textit{original amount deposited}\dotfill & \$16000\\ r=rate\to 8\%\to \frac{8}{100}\dotfill &0.08\\ t=years\dotfill &3 \end{cases} \\\\\\ A=16000[1+(0.08)(3)] \implies A = 19840~\hfill \underset{monthly~payment}{\stackrel{19840~\div \stackrel{months}{36}}{\approx \text{\LARGE 551.11}}}[/tex]

now let's check with the Credit Card

[tex]~~~~~~ \textit{Compound Interest Earned Amount} \\\\ A=P\left(1+\frac{r}{n}\right)^{nt} \quad \begin{cases} A=\textit{accumulated amount}\\ P=\textit{original amount deposited}\dotfill &\$16000\\ r=rate\to 15\%\to \frac{15}{100}\dotfill &0.15\\ n= \begin{array}{llll} \textit{times it compounds per year} \end{array}\dotfill &12\\ t=years\dotfill &7 \end{cases}[/tex]

[tex]A=16000\left(1+\frac{0.15}{12}\right)^{12\cdot 7} \implies A \approx 45425.81~\hfill \underset{monthly~payment}{\stackrel{45425.81~\div \stackrel{months}{84}}{\approx\text{\LARGE 540.78}}}[/tex]

well, seems the Credit Card is the better deal monthly wise, though in the long run is a lot of dough.

A plan which would give Genesis the lowest monthly payment is the monthly payment on a bank loan would be $317.50.

What is simple interest?

Mathematically, the simple interest earned can be calculated by using this formula:

A = P(1 + rt)

Where: A represents the future value.

P represents the principal or original value.

r represents the interest rate.

t represents the time.

Now Substituting the given parameters into the formula, we have,

A = 9000[1 + (0.09 × 3)]

A = 9000[1 + 0.27]

A = 9000 × 1.27

Future value, A = $11,430.

Next, we will calculate the monthly payment over a period of 36 months as follows:

Monthly payment = Future value/Time

Monthly payment = 11,430/36

Monthly payment = $317.50.

Similarly, we will calculate the compound interest earned on the credit card :

A = 9000[1 + (0.18/12)]⁽¹² ˣ ⁷⁾

A = 9000[1 + 0.015]⁸⁴

A = 9000 × 1.015⁸⁴

A = 9000 × 3.49258954

Future value, A = $31,433.31.

Next, we will calculate the monthly payment over a period of 84 months as follows;

Monthly payment = Future value/Time

Monthly payment = 31,433.31/84

Monthly payment = $372.21.

In this context, A plan which would give Genesis the lowest monthly payment is the monthly payment on a bank loan would be $317.50.

Read more on monthly payment here:

https://brainly.com/question/28660364

#SPJ1

please help me with this problem

Answers

Solution

In a cyclic quadrilateral, opposite angles is supplementary.

=> 3x - 59 + 2x - 1 = 180

=> 5x - 60 = 180

=> 5x = 180 + 60

=> 5x = 240

=> x = 48

Angle A = 2(48) + 4 = 100

Also, A + C = 180

=> C = 180 - 100

=> C = 80

MIRA 5(=2)-3(x-2) 5x=10-3x + 6 2x=4Mira and Vernon are simplifying the expression 5(x-2)-3(x-2). Who's process is correct, Mira, Vernon, or both?

Answers

Answer:

Mira is correct

Step-by-step explanation:

She did all the steps and Vernon skipped some so u dont rlly understand what he did u could lose points if it was an exam for example

Ana runs 3/4 mile in 6 minutes. Fill in the missing values in the table. Using the table, find the constant of proportionality. Then write the equation to represent this relationship.

Answers

1. The constant of proportionality when Ana runs 3/4 mile in 6 minutes is 0.125.

2. The missing value in the table will be 7.5 miles and 8.75 miles.

What is constant of proportionality?

It should be noted that the constant of proportionality simply illustrates the ratio of two given values that have a proportional relationship.

1. In this case, Ana runs 3/4 mile in 6 minutes. Therefore, the constant of proportionality will be:

y = kx

where y = distance

x = time

3/4 = 6x

0.75 = 6k

Divide

k = 0.75 / 6

k = 0.125

The constant of proportionality is 0.125.

2. The missing value in the table will be:

y = kx

when x = 60.

y = 60 × 0.125 = 7.5 miles.

y = kx

when x = 70

y = 70 × 0.125 =8.75 miles.

Learn more about constant of proportionality on

brainly.com/question/28413384

#SPJ1

Classify the equation 4x+2=4x+2 as having one solution,no solution or infinite many solutions.

Answers

Answer:

Infinite many solutions

Step-by-step explanation:

4x + 2 = 4x + 2

-4x         -4x

----------------------

2 = 2

or

4x + 2 = 4x + 2

      -2          -2

-----------------------

4x = 4x

Infinite many solutions

I hope this helps!

I don’t under what it is trying to say

Answers

Answer:

-2

Step-by-step explanation:

It’s asking you to solve the expression with the values given. After doing that, it’s asking you to simplify the result.


Expression - Does not have an equal sign, only the operation (multiplication, division, addition, subtraction).

The following table shows inflation over the given period.
Time span
1970-2010
1980-2010
Inflation
462%
164%
The following table shows average house priced for the given year:
Year
Average
home sale
1970
1980
$18,300
1990
1990-2010
67%
2000
2000-2010
26%
2010
$47,200
$79,100
$119,600
price
a) Find the percentage change in the home sale price between the year 1970 and 2010 using the above table
$156,000

Answers

The percentage change in the home sale price between the year 1970 and 2010 is 13.01%

What is percentage?

It is a relative value used to denote hundredths of any quantity. One percent (symbolized as 1%), being the hundredth component, is represented as 100 percent, while 200 percent signifies twice the amount specified. A number's percentage is its weight in relation to 100. For example in the case, there are 26 females and 24 boys in a class. 52% of the class is made up of girls, which means that out of 100 students, 52 are female.

Given Data

Average home sale

1980 = $18300

1990 - 2010= $2000

Percentage sale = [tex]\frac{20300}{156000}[/tex] × 100

Percentage sale = 13.01

The percentage change in the home sale price between the year 1970 and 2010 is 13.01%

To learn more about percentage, visit:

https://brainly.com/question/28269290

#SPJ9

What is the special angle pair relationship between ∠1 and ∠3?

Answers

Answer:

see explanation

Step-by-step explanation:

∠ 1 and ∠ 3 are corresponding angles

each is in the same position ( the upper left hand corner) in its group of 4 angles.

( Please help I'll mark brainliest )
Answer the question in the photo.

Answers

Answer: 5/3

Step-by-step explanation:

The scale factor is the same as the ratio of the length of a side of the image to the length of the corresponding side in the preimage.

We know that N'Q'=10 and NQ=6, so the scale factor is 10/6 = 5/3.

an elliptical arch is constructed which is 6 feet wide at the base and 9 feet tall in the middle. find the height of the arch exactly 1 foot in from the base of the arch.

Answers

The height of the arch is about 6.71 feet when measured from its base at a distance of one foot.

This is further explained below.

What is an ellipse?

Generally, An ellipse is a collection of points whose distances from a fixed point (focus) or fixed line (directrix) is constant and less than 1.

Depending on whether the transverse axis is horizontal or vertical, we may construct an equation for the ellipse. The main axis is located along the transverse axis. The ellipse may be represented using the following equations:

[tex]\begin{aligned}&\frac{(x-h)^2}{a^2}+\frac{(y-k)^2}{b^2}=1 \\&\frac{(y-k)^2}{a^2}+\frac{(x-h)^2}{b^2}=1\end{aligned}[/tex]

An elliptical arch is described as having a center height of 9 feet and a width of 6 feet. The precise height of the arch from the base has to be determined.

[tex]\frac{(y-k)^2}{a^2}+\frac{(x-h)^2}{b^2}=1[/tex]

The center (h, k) is where the graph is since we centered it at the origin (0,0). We only need to enter the numbers into the equation at this point.

[tex]\begin{aligned}\frac{(y-0)^2}{9^2}+\frac{(x-0)^2}{3^2} &=1 \\\frac{y^2}{81}+\frac{x^2}{9} &=1\end{aligned}[/tex]

Since we divided the arch in half, 0 [tex]\leq[/tex] 9 in this instance. All that is left to do is calculate the arch's height in feet from the base.

The height of 1 foot from each end of the arch must thus be determined. The locations of the arch's ends are x=-3 and x=3. Here, we have a choice between the two. We may choose x=3-1=2 for this. Therefore, all that is left to do is enter x=2 in our usual form and work out the value of y that results.

[tex]\frac{y^2}{81}+\frac{(2)^2}{9}=1[/tex]

y^2+36=81

y^2+36-36=81-36

y=3√5

Therefore, the height of the arch at 1ft from the base is 3 √5ft or approximately 6.71ft

Read more about height

https://brainly.com/question/10726356

#SPJ1

8. Order these numbers from least to greatest. 26 087, 2.6 x 10', 26 079, 2.6432 x 10,26 295, 2.6803 x 10​

Answers

The numbers are  26.087, 2.6 x 10', 26.079, 2.6432x 10,26.295, 2.6803x10​. The order of the number from least to greatest is 26, 26.079, 26.087, 26.295, 26.432, 26.803.

Given that,

The numbers are  26.087, 2.6 x 10', 26.079, 2.6432x 10,26.295, 2.6803x10​.

We have to find the order of the numbers from least to greatest.

The order of the numbers from least to greatest is nothing but an ascending order.

Ascending order is nothing but ascending order refers to the grouping of information from the lowest to the highest value. Increasing order is another name for it.

Now, we take the numbers

1st number is 26.087.

2nd number is 2.6×10=26.

3rd number is 26.079.

4th number is 2.6432×10=26.432.

5th number is 26.295.

6th number is 2.6803×10=26.803.

Now, we write in ascending order

26, 26.079, 26.087, 26.295, 26.432, 26.803.

Therefore, the order of the number from least to greatest is 26, 26.079, 26.087, 26.295, 26.432, 26.803.

To learn more about numbers visit: https://brainly.com/question/17429689

#SPJ9

Henry has $75 in his bank account. If he withdraws $75 from the account, how much money will be left in the account?

Answers

Answer:

$0

Step-by-step explanation:

Henry has $75 in his bank account. If he withdraws $75 from the account, how much money will be left in the account?

$75 beginning balance - $75 withdrawal = $0

Henry would have $0 left in his bank account.

will give brainliest

Write the set of coordinates for a reflection of the figure over the x-
axis.
(-1, 3) (-3, 1) (-5, 1) (-4, 6)

Answers

Check the picture below.

Find a point-slope form for the line with slope = 1/5 and passing through the point
(- 2, -3).
The equation of the line in point-slope form is…

Answers

Answer:

y+3= 1/5 (x+2)

Step-by-step explanation:

The point slope form looks like this.

y - y1 = m (x - x1)

the m represent slope and (x1, y1) are the coordinates of the point that the line passes through.

a rope 18M and 50Cm is to be divided into four equal parts. how long will each part be? give your answer in metres and centimetres

Answers

Answer:

4 m and 62.5 cm

Step-by-step explanation:

1 meter = 100 cm

So 18m and 50 cm = 18 x 100 + 50 cm = 1850 cm

If it is going to be divided into 4 equal parts, each part = 1850/4 = 462.5 cm

462.5 cm = 462.5/ 100 = 4 m and 0.625 m

0.625m = 0.625 x 100 = 62.5 cm

So each piece is 4 m and 62.5 cm

Type the next number in the list
3, 12, 60, 240, 1200 what’s the next number

Answers

In the given number series 3, 12, 60, 240, 1200, the next number is 4800.

How to find the next number?

3, 12, 60, 240, 1200

Solution:

In the given sequence,

3, 12, 60, 240, 1200

Each intermittent number is multiplied by 4 and the number next to it by 5.

3*4 = 12

12*5 = 60

60*4 = 240

240*5 = 1200

1200*4 = 4800

Therefore, the next number is 4800.

To learn more about number series sums, refer

https://brainly.com/question/26748083

#SPJ13

I’m trying to figure out this question but can’t need help asap

Answers

Answer:

bottom right rectangle: 5x − y

bottom circle: 6x

Step-by-step explanation:

First, solve for the bottom right rectangle by finding the difference between the top right rectangle and the middle right circle:

(9x + 4y) − (4x + 5y)

= (9x − 4x) + (4y − 5y)

= 5x − y

Next, to find the bottom circle, add the bottom left rectangle with the bottom right rectangle (the one we just solved for):

(x + y) + (5x − y)

= (x + 5x) + (y − y)

= 6x


What is the slope of the line?

Answers

Answer:

Step-by-step explanation:

Slope= rise/run or y2-y1/x2-x1

the two points are (4,-4) and (1,4)

so, it is (( -4) - 4) /(4-1)= -8/3

slope = -8/3

The population of the United States is approximately 3.28×108 people, and the population of Brazil is approximately 2.12×108 people. What is the difference between the populations of the United States and Brazil?

Answers

Answer:

[tex]1.16 \times {10}^{8} [/tex]

Step-by-step explanation:

[tex](3.28 \times {10}^{8} ) - (2.12 \times {10}^{8} ) = 1.16 \times {10}^{8} [/tex]

the circumference of a circle is 60cm what is the area
step by step explanation please

Answers

The area of a circle with the value of its circumference as 60cm is 286. 38cm²

How to determine the area of the circle

The formula for determining the formula for circumference of  a circle is expressed as;

Circumference = 2 πr

Given that;

π  has a fractional value of 22/ 7 and a decimal value of 3. 14r is the radius of the circle

Now, we have to substitute the value of circumference of the circle as 60cm

We have;

60 = 2 πr

Now, substitute the value for π into the equation, we have;

60 = 2× 3.14r

Multiply the values, we get;

60 = 6.28r

Make 'r' the subject of formula in the equation

r = 60/6.28

r = 9.55cm

But we know that e formula for area of a circle is expressed as;

Area = πr²

Now, substitute the value of radius into the formula, we have;

Area = 3. 14(9.55)²

Find the squares

Area = 3.14(91.2025)

Multiply the values by expanding the bracket

Area = 286. 38cm²

Hence, the area is 286. 38cm²

Learn more about area here:

https://brainly.com/question/25292087

#SPJ1

if the die were loaded so that the face with the 3 on it were twice as likely to come up as each of the other five faces, would this change the value of p(odd number)? explain.

Answers

if the die were loaded so that the face with the 3 on it were twice as likely to come up as each of the other five faces, No , it does not  change the value of p(odd number) .

a) The sample space is the set of all possible outcomes. By definition, the elements of a set should not be repeated. Hence, the sample space S = {1, 2, 3}

However, the sample is not equiprobable because each element has different probabilities.

b) P(odd number) = number of odd digits / number of faces = 4/6=2/3

Note that the odd numbers are 1 (on three faces) and 3 (on one face).

c) The fact the die has been biased does not change the possible outcomes. It only changes the probability of getting any given number.

d) Because the 3-face has been loaded, this probability changes. In fact, it is calculated thus:

Let's assume the probability for 1 or 2 is . Then that of 3 is (because it is twice the others). The sum of probabilities must be 1.

x+x+x+x+x+2x=1

7x=1

x=1/7

P(odd number) = 3 * Prob(1) + Prob(3)

=3*1/7 +2/7 = 5/7

learn more about of dice here

https://brainly.com/question/14606607

#SPJ4

A certain Figure Q is enlarged to form Figure P. Are Figures Q and P congruent?

Answers

[tex]\quad \huge \quad \quad \boxed{ \tt \:Answer }[/tex]

[tex]\qquad \tt \rightarrow \:Not \:\: Congruent [/tex]

____________________________________

[tex] \large \tt Solution \: : [/tex]

If a figure is enlarged, it will still be similar to the original figure, but it won't be congruent to the original figure, because, to be congruent the figures should be of equal size/ coincide each other perfectly, which isn't possible in this scenario.

Answered by : ❝ AǫᴜᴀWɪᴢ ❞

A $100 sweatshirt is put on sale for 40% off. One week later, the sale price is marked up by 40%. Will the marked-up price of the sweatshirt be less than, greater than, or equal to the initial price of $100?

Answers

The new price after being marked up is less than initial price of $100.

The price of sweatshirt after discount = 100 - 40%×100

Converting percentage to fraction

The price of sweatshirt after discount = 100 - (40/100)×100

The price of sweatshirt after discount = 100 - 40

Performing subtraction

The price of sweatshirt after discount = $60

The marked up sale price = 60 + 40%×60

Converting percentage to fraction

The marked up sale price = 60 + (40/100)×60

Cancelling zero and performing multiplication

The marked up sale price = 60 + 24

Performing addition

The marked up sale price = $84

Thus, the marked up price after discount will be less than the initial price of $100.

Learn more about Percentage -

https://brainly.com/question/24304697

#SPJ1

TABLE-TENNIS TABLES Target purchases TIGA table-tennis tables at a cost of $180 each. If the company's operating expenses are 16% of cost, and a net profit of 7% of cost is desired, find the selling price of one table-tennis table.​

Answers

Answer:

The selling price of one table-tennis is $221.40

Step-by-step explanation:

3(x+y)=y
If (x,y) is a solution to the question above and
Y= 0, what is the ratio x/y ?
A) -4/3
B) -2/3
C) 1/3
D) 2/3

Answers

The value of x / y in the given equation will be -2 / 3. The correct option is B.

What is an expression?

The mathematical expression combines numerical variables and operations denoted by addition, subtraction, multiplication, and division signs.

Mathematical symbols can be used to represent numbers (constants), variables, operations, functions, brackets, punctuation, and grouping. They can also denote the logical syntax's operation order and other properties.

Given expression is 3(x+y) = y, the value of x / y will be calculated as:-

3(x+y) = y

( x + y ) / y = 1 / 3

( x / y ) + 1 =  1 / 3

( x / y ) = ( 1 / 3 ) - 1

( x / y ) =  -2 / 3

Therefore, the value of x / y in the given equation will be -2 / 3. The correct option is B.

To know more about an expression follow

https://brainly.com/question/28001079

#SPJ1

cual es el volumen de una pila que mide 100 cm de ancho por 110 de largo por 130 cm de alto

Answers

The volume of the pile based on the width, height, and length of the pile can be found to be 1.43 m³

How to find the volume?

The volume of the pile can be found by the formula:

= Length x Width x Height

From the question, the dimensions of the pile are:

Length = 110 cm

Width = 100 cm

Height = 130 cm

The volume is:

= 110 x 100 x 130

= 1,430,000 cm³

Converted to meters gives:

= 1,430,000 / 1,000,000

= 1.43 m³

In conclusion, the volume in meters of the pile is 1.43 m³

Find out more on volume at https://brainly.com/question/1972490

#SPJ1

kareem needs to make a total of 40 deliveries this week. so far hes completed 28 of them. what percentage of deliveries has he completed?


thank youuuu

Answers

If Kareem has completed 28 out of the 40 deliveries he needs to fulfill this week, then he has completed 70 percentage of his total deliveries.

As per the question statement, Kareem needs to make a total of 40 deliveries this week, and he has completed 28 thus far.

We are required to calculate the percentage of deliveries, Kareem has completed so far.

To solve this question, we simply need to divide the number of deliveries completed so far, by the total number of deliveries Kareem needs to fulfill in the week, and then multiply the quotient with 100, to obtain our desired answer.

Here, number of deliveries completed so far by Kareem  = 28,

And, total number of deliveries Kareem needs to fulfill = 40.

Therefore, percentage of deliveries, Kareem has completed so far

= [(28/40) * 100]

= [(7/10) * 100]

= (7 * 10)

= 70%.

Percentage: In Mathematics, percentage is the number or ratio, being expressed as a fraction of 100 and often denoted using the percent sign, "%".

To learn more about Percentage, click on the link below.

https://brainly.com/question/24159063

#SPJ1

Other Questions
Which nutrient-dense food contains both protein and monosaturated fat?A. beansB. avocadoC. cruciferous vegetablesD. kale Eight leaders and 143 members of a youth club go by bus to a bowling alley.The Beezer Bus Company has buses for hire that hold 32 passengers each.Each bus costs 140 to hire.The cost is split equaly between the members.The leaders do not pay.How much does each member need to pay to cover the cost of hiring the buses?....(3 m All are main food allergies except eggs sugar shellfish wheat someone help me asap ill give you 50 points Please select one of the following prompts to respond to in 200 words or less. 1) American social reformer, abolitionist, writer and statesman Frederick Douglass said, Some know the value of education by having it. I knew its value by not having it. Explain ways in which education has directly influenced you and your ability to do good in the world. How will you use the curricular flexibility and co-curricular opportunities of the University of Rochester experience to grow and to promote positive change for yourself, your community and the world?2) Dr. Donna Strickland, University of Rochester alum and 2018 Nobel Prize winner in Physics said, Theres no point in me being anything other than me. The University of Rochester encourages each student to embrace who they are and create their own individual curricular path and experience. How will you use the opportunities here to fully be who you are? What unique perspectives will you bring to our community?3) Susan B. Anthony, champion of abolition and womens rights, once said Organize, agitate, educate must be our war cry. As you look to join our community of doers and disruptors, in what ways do you envision using both the curricular flexibility and co-curricular opportunities to invoke change for marginalized groups?4) The University of Rochester benefactor entrepreneur, photography pioneer and philanthropist George Eastman said, What we do during our working hours determines what we have; what we do in our leisure hours determines what we are. Looking forward towards your college experience, what do you hope to do outside of the classroom at the University of Rochester that will enhance who you are as a person? How will specific academic and social opportunities here help you grow?Which short essay prompt above will you be responding to?*Frederick DouglassDr. Donna StricklandSusan B. AnthonyGeorge Eastman PLEASE HELP Create a scatter plot for the data below.Title the graph. Determine an appropriate scale for the x and y axis. Determine an appropriate interval for the x and y axis. Title the axes Write a paragraph supporting your opinion on the following question: Who played the most important role in exploring the Americas?(PLS ANSWER ASAP)a. Christopher Columbus b. Queen Isabella c. Amerigo Vespucci The speaker of Hughes's poem is successful in what he sets out to do at the start of the poem.True or False where are the Sonnet with bird three quatrains Today I was called to a particular conference at the residence of one of the bishops of this kingdom, along with many others of his rank and some other deputies from the nobility. There a certain formula of association was proposed, whose purpose was to break the last edict of pacification and to place the king at war against those of the so-called Protestant religion. . . . I remarked that it seemed to me that the king ought not to be counseled to go to war, because of the recent injuries the kingdom has suffered, and that anyone who favors civil war is ungodly and deserves our prayer. . . . Concerning the present formula of association, I say that I cannot sign it and call myself a servant of the king. I know that war is unjust if it is not conducted and undertaken by the king in his faithful and holy council.a) Describe one historical development illustrated by the passage.b) Describe Blancheforts argument regarding war against the Protestants. Name:15 Hannah took a trip to visit her cousin. She drove120 miles to reach her cousin's house and the samedistance back home. It took her 1.2 hours to gethalfway to her cousin's house. What was heraverage speed, in miles per hour, for the first 1.2hours of the trip? Hannah's average speed for theremainder of the trip to her cousin's house was 40miles per hour. How long, in hours, did it take herto drive the remaining distance? Traveling homealong the same route, Hannah drove at an averagerate of 55 miles per hour. After 2 hours her carbroke down. How many miles was she from home? 7. Let A=(8,9,10,11,12,13).n=a. How many subsets does A have?b. How many proper subsets does A have?subsets.a. A has(Type a whole number.)b. A has(Type a whole number.)proper subsets. if three sides of an equilateral triangle measures 3X +12, 8X -3 and 5Y plus one what is the value of X and Y in the length of each side Which of the following has highest boiling points propanol or hexanone? Which statement best describes the Siege of Boston in 1775?O The British tried to capture two important hills.O The Continental army lost many soldiers to desertion. The British tried to take back the city from colonists.O Colonists tried to take back the city from the British. Find the measure of the missing angles. Select the correct answer.What is this expression in simplified form?52-96 in the first month of operations, the total of the debit entries to the cash account amounted to $1,200 and the total of the credit entries to the cash account amounted to $800. the cash account has a(n) . Scientists determined that Alaska's average winter was -30.2C. The difference between this temperature and Alaska's highest recorded temperature was 50.33 degrees. What was Alaskas highest recorded temperature?* Determine whether each number is rational or irrational. What are the reserved powers? Give some examples